- PowerScore Staff
- Posts: 5972
- Joined: Mar 25, 2011
- Thu Oct 29, 2020 5:34 pm
#80533
This game is also discussed in our Podcast: LSAT Podcast Episode 70: The May 2020 LSAT-Flex Logic Games Section
Complete Question Explanation
(The complete setup for this game can be found here: https://forum.powerscore.com/lsat/viewtopic.php?t=33050)
The correct answer choice is (B).
The question stem asks you to replace the fourth rule with a different rule that has an identical effect, with no additional effects. The fourth rule created a not-block between G and J, and thus we must seek to replicate that result in the answers. As is often the case with Rule Substitution questions, at a glance it's difficult to imagine what route the test makers might choose, but the best place to start is to determine what you know about each variable in the rule being suspended.
In this case, we know that G cannot play in the first game due to the first rule, and J cannot play in the third game due to the second rule. Thus, as Kelsey notes below, "the only game in which G and J could possibly be together is the 2nd game. So we don't really have to worry about if J is in the 1st game or if G is in the 3rd game. To keep them apart, we just have to show that they cannot both be in the 2nd game. That's what answer choice (B) gives us: if J is in the 2nd game, G is in the 3rd tells us that J and G cannot be together in the 2nd game which means they cannot be together in any game!"
Answer choice (A): This answer does force G and J apart when H plays in the first game. However, if H doesn't play in the first game, then G and J can play together in the second game (with F and H in the third game, and L and M in the first game). Thus, since this answer does not always keep G and J apart, it is incorrect.
Answer choice (B): This is the correct answer choice. This is an unusual correct answer since it directly addresses the exact two variables in the suspended rule. That's not normally how LSAC likes to make answer choices for these questions. However, as noted above, this answer separated G and J from being together in the second game—the only game where they could appear together—and is thus correct.
Answer choice (C): This rule creates a horizontal JF block. However, when J is second then G and J can play together in the second game (with F and H in the third game, and L and M in the first game). Thus, since this answer does not always keep G and J apart, it is incorrect.
Answer choice (D): This rule creates a horizontal JG block. This clearly meets the condition of keeping G and J apart, and so it appears promising at first. However, it also creates additional effects that go beyond the original fourth rule. For example, one of G or J always plays in the second game now, which was not an original condition.
Note as well how similar this answer is to (B). Answer choice (B) creates this horizontal block relationship just when J plays in the second game, whereas this answer creates this relationship when J plays in the first and second games. That extra difference—J playing the first game—overextends this answer and makes it incorrect.
Answer choice (E): This answer is immediately suspect since Template #4 allowed G and L to play in the same game. And that problem is enough to eliminate the answer.
Additionally, this rule still allows G and J to play together in the second game (with F and H in the third game, and L and M in the first game). Thus, since this answer does not always keep G and J apart, it is incorrect for that reason as well.
Complete Question Explanation
(The complete setup for this game can be found here: https://forum.powerscore.com/lsat/viewtopic.php?t=33050)
The correct answer choice is (B).
The question stem asks you to replace the fourth rule with a different rule that has an identical effect, with no additional effects. The fourth rule created a not-block between G and J, and thus we must seek to replicate that result in the answers. As is often the case with Rule Substitution questions, at a glance it's difficult to imagine what route the test makers might choose, but the best place to start is to determine what you know about each variable in the rule being suspended.
In this case, we know that G cannot play in the first game due to the first rule, and J cannot play in the third game due to the second rule. Thus, as Kelsey notes below, "the only game in which G and J could possibly be together is the 2nd game. So we don't really have to worry about if J is in the 1st game or if G is in the 3rd game. To keep them apart, we just have to show that they cannot both be in the 2nd game. That's what answer choice (B) gives us: if J is in the 2nd game, G is in the 3rd tells us that J and G cannot be together in the 2nd game which means they cannot be together in any game!"
Answer choice (A): This answer does force G and J apart when H plays in the first game. However, if H doesn't play in the first game, then G and J can play together in the second game (with F and H in the third game, and L and M in the first game). Thus, since this answer does not always keep G and J apart, it is incorrect.
Answer choice (B): This is the correct answer choice. This is an unusual correct answer since it directly addresses the exact two variables in the suspended rule. That's not normally how LSAC likes to make answer choices for these questions. However, as noted above, this answer separated G and J from being together in the second game—the only game where they could appear together—and is thus correct.
Answer choice (C): This rule creates a horizontal JF block. However, when J is second then G and J can play together in the second game (with F and H in the third game, and L and M in the first game). Thus, since this answer does not always keep G and J apart, it is incorrect.
Answer choice (D): This rule creates a horizontal JG block. This clearly meets the condition of keeping G and J apart, and so it appears promising at first. However, it also creates additional effects that go beyond the original fourth rule. For example, one of G or J always plays in the second game now, which was not an original condition.
Note as well how similar this answer is to (B). Answer choice (B) creates this horizontal block relationship just when J plays in the second game, whereas this answer creates this relationship when J plays in the first and second games. That extra difference—J playing the first game—overextends this answer and makes it incorrect.
Answer choice (E): This answer is immediately suspect since Template #4 allowed G and L to play in the same game. And that problem is enough to eliminate the answer.
Additionally, this rule still allows G and J to play together in the second game (with F and H in the third game, and L and M in the first game). Thus, since this answer does not always keep G and J apart, it is incorrect for that reason as well.
Dave Killoran
PowerScore Test Preparation
Follow me on X/Twitter at http://twitter.com/DaveKilloran
My LSAT Articles: http://blog.powerscore.com/lsat/author/dave-killoran
PowerScore Podcast: http://www.powerscore.com/lsat/podcast/
PowerScore Test Preparation
Follow me on X/Twitter at http://twitter.com/DaveKilloran
My LSAT Articles: http://blog.powerscore.com/lsat/author/dave-killoran
PowerScore Podcast: http://www.powerscore.com/lsat/podcast/